Proofs Homework: Explain Why m+n≠10 Example Not Sufficient

  • Thread starter Thread starter ver_mathstats
  • Start date Start date
  • Tags Tags
    Example Proofs
Click For Summary
The discussion clarifies that providing a single example, such as m=1 and n=2, is insufficient to prove the statement m+n≠10 for all integers. This is because a single counterexample does not account for other integer combinations that could sum to 10, like m=4 and n=6. To prove a universal statement, one must demonstrate that no integers exist that satisfy the equation m+n=10, rather than just showing that some do not. The inability to prove the statement means it cannot be sufficiently supported by a single example. Thus, a comprehensive approach is necessary to address universal quantifiers in mathematical proofs.
ver_mathstats
Messages
258
Reaction score
21

Homework Statement


Suppose you are asked to prove that for all integers m and n, m+n≠10. You give the example m=1 and n=2. Why is this not sufficient?

Homework Equations

The Attempt at a Solution


I can't quite understand why it is not sufficient? Could someone please explain to me why is it insufficient?
 
Physics news on Phys.org
ver_mathstats said:

Homework Statement


Suppose you are asked to prove that for all integers m and n, m+n≠10. You give the example m=1 and n=2. Why is this not sufficient?

Homework Equations

The Attempt at a Solution


I can't quite understand why it is not sufficient? Could someone please explain to me why is it insufficient?
What do you mean? ##m+n\neq 10## isn't true for some integers. It is sufficient to say ##4+6=10## in order to disprove it. As it cannot be proven, there cannot be a sufficient condition for a proof either.

An example is never sufficient to prove an all quantifier, only to disprove one.
 
Put slightly differently, you are asked to show that no integers n and m exist that satisfy n+m = 10. Then it is not enough to give a single example of integers that do not sum to 10 as another pair of integers could do that (and does).
 
Question: A clock's minute hand has length 4 and its hour hand has length 3. What is the distance between the tips at the moment when it is increasing most rapidly?(Putnam Exam Question) Answer: Making assumption that both the hands moves at constant angular velocities, the answer is ## \sqrt{7} .## But don't you think this assumption is somewhat doubtful and wrong?

Similar threads

  • · Replies 3 ·
Replies
3
Views
1K
  • · Replies 2 ·
Replies
2
Views
2K
  • · Replies 3 ·
Replies
3
Views
2K
Replies
5
Views
2K
  • · Replies 2 ·
Replies
2
Views
2K
  • · Replies 2 ·
Replies
2
Views
2K
  • · Replies 8 ·
Replies
8
Views
9K
  • · Replies 10 ·
Replies
10
Views
2K
  • · Replies 7 ·
Replies
7
Views
2K
  • · Replies 6 ·
Replies
6
Views
3K